Difference between revisions of "2014 AMC 10A Problems/Problem 7"

m (Solution 2)
m (Problem)
Line 14: Line 14:
 
\textbf{(IV)}\ \frac{x}{y} < \frac{a}{b}</math>
 
\textbf{(IV)}\ \frac{x}{y} < \frac{a}{b}</math>
  
<math> \textbf{(A)}\ 0\qquad\textbf{(B)}\ 1\qquad\textbf{(C)}\ 2\qquad\textbf{(D)}}\ 3\qquad\textbf{(E)}\ 4</math>
+
<math> \textbf{(A)}\ 0\qquad\textbf{(B)}\ 1\qquad\textbf{(C)}\ 2\qquad\textbf{(D)}\ 3\qquad\textbf{(E)}\ 4</math>
  
 
==Solution==
 
==Solution==

Revision as of 20:12, 23 April 2015

Problem

Nonzero real numbers $x$, $y$, $a$, and $b$ satisfy $x < a$ and $y < b$. How many of the following inequalities must be true?

$\textbf{(I)}\ x+y < a+b\qquad$

$\textbf{(II)}\ x-y < a-b\qquad$

$\textbf{(III)}\ xy < ab\qquad$

$\textbf{(IV)}\ \frac{x}{y} < \frac{a}{b}$

$\textbf{(A)}\ 0\qquad\textbf{(B)}\ 1\qquad\textbf{(C)}\ 2\qquad\textbf{(D)}\ 3\qquad\textbf{(E)}\ 4$

Solution

First, we note that $\textbf{(I)}$ must be true by adding our two original inequalities. \[x<a, y<b\] \[\implies x+y<a+b\]

Though one may be inclined to think that $\textbf{(II)}$ must also be true, it is not, for we cannot subtract inequalities.

In order to prove that the other inequalities are false, we only need to provide one counterexample. Let's try substituting \[x=-3,y=-2,a=1,b=1\]

$\textbf{(II)}$ states that $x-y<a-b \implies -3-(-2)<1-1 \implies 1<0$ Since this is false, $\textbf{(II)}$ must also be false.

$\textbf{(III)}$ states that $xy<ab \implies (-3)(-2)<1\cdot 1 \implies 6<1$. This is also false, thus $\textbf{(III)}$ is false.

$\textbf{(IV)}$ states that $\frac{x}{y}<\frac{a}{b} \implies \frac{-3}{-2}<\frac{1}{1}\implies 1.5<1$. This is false, so $\textbf{(IV)}$ is false.

One of our four inequalities is true, hence, our answer is $\boxed{\textbf{(B) 1}}$

Solution 2

Also, with some intuition, we could have plugged $0=X$, $1=A$, $-3=Y$, and $-2=B$ and then plugged these values into the equations to see which ones held.

See Also

2014 AMC 10A (ProblemsAnswer KeyResources)
Preceded by
Problem 6
Followed by
Problem 8
1 2 3 4 5 6 7 8 9 10 11 12 13 14 15 16 17 18 19 20 21 22 23 24 25
All AMC 10 Problems and Solutions

The problems on this page are copyrighted by the Mathematical Association of America's American Mathematics Competitions. AMC logo.png